Which option is correct and how would one solve for it?

Which Option Is Correct And How Would One Solve For It?

Answers

Answer 1

Answer:

28

Step-by-step explanation:

We need to find the value of [tex]\Sigma_{x=0}^3\ 2x^2[/tex]

We know that,

[tex]\Sigma n^2=\dfrac{n(n+1)(2n+1)}{6}[/tex]

Here, n = 3

So,

[tex]\Sigma n^2=\dfrac{3(3+1)(2(3)+1)}{6}\\\\\Sigma n^2=14[/tex]

So,

[tex]\Sigma_{x=0}^3\ 2x^2=2\times 14\\\\=28[/tex]

So, the value of [tex]\Sigma_{x=0}^3\ 2x^2[/tex] is 28. Hence, the correct option is (d).


Related Questions

football team, won 35 out of 39 games over a period of 4 years. if they keep winning pace, predict how many games you would expect them to win over the next 78 football games

Answers

Answer:

70

Step-by-step explanation:

If the team continues with same pace, they expected wins as per previous ratio:

35/39*78 = 70

Expected wins 70 out of 78 games

In order to earn an A in her math course,
Bernadette must have an average of at
least 90 on her exam scores. She has
grades of 83, 97, 89, and 82 on her first 4
exams. What is the minimum she can
score on the final exam to earn an A in the
course?​

Answers

Step-by-step explanation:

Let minimum score on the final exam to earn an A be X

[tex]mean \: = \frac{sum \: of \: observation}{number \: of \: observation} [/tex]

[tex]90 = \frac{83 + 97 + 89 + 82 + x}{5} [/tex]

Further solving :

X = 99 marks

At Jefferson Middle School, eighty-two students were asked which sports they plan to participate in for the coming year. Twenty students plan to participate in track and cross country; six students in cross country and basketball; and eight students in track and basketball. Twelve students plan to participate in all three sports. A total of thirty students plan to participate in basketball, and a total of forty students plan to participate in cross country. Ten students don't plan to participate in any of the three sports. How many students plan to just participate in cross country? 2 4 40 30

Answers

Answer:

40

Step-by-step explanation:

In the question only lies the answer:

"and a total of forty students plan to participate in cross country."

Answer:

2

Step-by-step explanation:

2

PLEASE HELP!!
Solve for y
a) 8
b) 12
c) 3V7
d) 4V7

Answers

Answer:

C. [tex] y = 3\sqrt{7} [/tex]

Step-by-step explanation:

Based on the right triangle altitude theorem, the altitude, y, in the diagram above, equals the geometric mean of 9 and 7.

This implies => [tex] y = \sqrt{9*7} [/tex]

Thus, solve for y.

[tex] y = \sqrt{9} * \sqrt{7} [/tex]

[tex] y = 3\sqrt{7} [/tex]

The answer is C. [tex] y = 3\sqrt{7} [/tex]

Evaluate the expresión 6c-d when c=2 and d=10 I need help?

Answers

Answer:

the answer is 18

Step-by-step explanation:

8 is the answer

If an image of a triangle is congruent to the pre-image, what is the scale factor of the dilation? 0.1 1 10

Answers

Answer:

1

Step-by-step explanation: diliation is like multiplilcation if you were to do 3*1 =3. simply congruent means all sides and angles are the same.  

Given that the image and the preimage of the triangle are congruent, their

dimensions are the same.

The scale factor of dilation of an image of a triangle that is congruent to the pre-image is; 1

Reasons:

Let ΔABC represent the preimage, and let ΔA'B'C' represent the image.

Given that the image and the preimage are congruent, we have;

AB ≅ A'B'

BC ≅ B'C'

AC ≅ A'C'

By definition of congruency, we have;

AB = A'B'

BC = B'C'

AC = A'C'

The scale factor of dilation is given as follows;

[tex]\displaystyle Scale \ factor = \mathbf{ \frac{A'B'}{AB}} = \frac{AB}{AB} = 1[/tex]

Therefore;

If the image is congruent to the pre-image, the scale factor of dilation is; 1

Learn more about dilation transformation here:

https://brainly.com/question/5453159

Given the graph, find an equation for the parabola.

Answers

Answer:

[tex]\Large \boxed{\sf \bf \ \ y=\dfrac{1}{16}(a-3)^2-2 \ \ }[/tex]

Step-by-step explanation:

Hello, please consider the following.

When the parabola equation is like

[tex]y=a(x-h)^2+k[/tex]

The vertex is the point (h,k) and the focus is the point (h, k+1/(4a))

As the vertex is (3,-2) we can say that h = 3 and k = -2.

We need to find a.

The focus  is (3,2) so we can say.

[tex]2=-2+\dfrac{1}{4a}\\\\\text{*** We add 2. ***}\\\\\dfrac{1}{4a}=2+2=4\\\\\text{*** We multiply by 4a. ***}\\\\16a=1\\\\\text{*** We divide by 16. ***}\\\\a=\dfrac{1}{16}[/tex]

So an equation for the parabola is.

[tex]\large \boxed{\sf y=\dfrac{1}{16}(a-3)^2-2 }[/tex]

Hope this helps.

Do not hesitate if you need further explanation.

Thank you

Find the value of x.
A. 3
B. 9
C. 0
D. 12

Answers

Answer:

x=3

Step-by-step explanation:

(segment piece) x (segment piece) =   (segment piece) x (segment piece)

3x(x+1) = 4x(x)

Divide each side by x

3x(x+1)/x = 4x(x)/x

3(x+1) = 4x

Distribute

3x+3 = 4x

Subtract 3x

3x+3-3x= 4x-3x

3 =x

Answer:

x = 3

Step-by-step explanation:

0 is a rediculas answer 9 and 12 are to big.

The lines are supposed to have a simular length:

3(3) + 4 = 13

4(3) + 3 = 15

These are the best answers that fit.

Please help me with this question

Answers

Answer:

  0 ≤ x ≤ 10

Step-by-step explanation:

The domain of f(x) is the set of values of x for which the function is defined. Here, the square root function is only defined for non-negative arguments, so we require ...

  -x^2 +10x ≥ 0

  x(10 -x) ≥ 0

The two factors in this product will both be positive only for values ...

  0 ≤ x ≤ 10 . . . . the domain of f(x)

Which of the following is the graph of the quadratic parent function

Answers

Answer: Choice C

This is the graph of y = x^2. It is a parabola that opens upward and has its vertex at the origin. Applying various transformations to the parent function will allow us to produce any parabolic graph we want. In effect, the parent function is like the most basic building block.

Candice spent 5 1/4 hours doing her homework. Her brother, Ronald, spent 1/2 that number of hours doing his homework. How many hours did Ronald spend on his homework?

Answers

Answer:

Step-by-step explanation:

½ of 5¼

½×(21/4)

=21/8

=2⅝ hours

Answer:

2 5/8

Step-by-step explanation:

you would divide 5 1/4 by 2 :

5 divided by 2 =2 1/2

1/4 divided by 2=1/8

then make the numbers have the same denomanator

1/2, 2/4, 4/8

1/8,

then you add

2 4/8+1/8=2 5/8

I NEED HELP WITH THESE 4 ASAP

Answers

Answer:

I'm confused by this. What do they mean by prove?

Step-by-step explanation:

The graph of the function f(x) = (x − 3)(x + 1) is shown.

On a coordinate plane, a parabola opens up. It goes through (negative 1, 0), has a vertex at (1, negative 4), and goes through (3, 0).
Which describes all of the values for which the graph is positive and decreasing?

all real values of x where x < −1
all real values of x where x < 1
all real values of x where 1 < x < 3
all real values of x where x > 3

Answers

Answer:

  x < -1

Step-by-step explanation:

Since the parabola opens upward, it is positive and decreasing where the left branch is above the x-axis: all points to the left of x=-1.

  all real values of x where x < -1

Quadrilateral RSTV is dilated with respect to the origin by a scale factor of 1.5 to produce quadrilateral R'S'T'V' . Vertex R is located at (6, -9). Which ordered pair represents R' after the dilation?

Answers

Answer:

(9, -13.5)

Step-by-step explanation:

It's given in the question that a quadrilateral RSTV is dilated with a scale factor of 1.5 with respect to the origin to form R'S'T'V'.

Rule for dilation is,

(x, y) → (kx, ky)

where 'k' is the scale factor.

If vertex R of the quadrilateral is (6, -9),

By the given rule of dilation,

R(6, 9) → R'[(1.5 × 6), -(1.5 × 9)]

           → R'(9, -13.5)

Therefore, Option given in bottom right (9, -13.5) will be the answer.

Billy has x marbles. Write an expression for the number of marbles the following have… a) Charlie has 5 more than Billy b) Danny has 8 fewer than Billy c) Eric has three times as many as Billy

Answers

Answer:

Charlie: 5 + xDanny: x - 8Eric: x × 3

5(y–3.8)=4.7(y–4) help help

Answers

Answer:

y = 2/3 or 0.667

Step-by-step explanation:

5(y–3.8)=4.7(y–4)

Expand the terms in the bracket

That's

5y - 19 = 4.7y - 18.8

Group like terms

5y - 4.7y = 19 - 18.8

0.3y = 0.2

Divide both sides by 0.3

We have the final answer as

y = 2/3 or 0.667

Hope this helps you

Lisa built a rectangular flower garden that is 4 meters wide and has a perimeter of 26 meters.
What is the length of Lisa's flower garden?

Answers

Answer:

9 m

Step-by-step explanation:

Given that

Width of rectangular flower garden, w = 4 m

Perimeter of rectangular flower garden, p = 26 m

To find:

Length of Lisa's flower garden = ?

Solution:

First of all, let us understand perimeter, length and width of a rectangle.

Let ABCD be a rectangle. Please refer to the attached image.

Opposite sides of a rectangle are equal to each other.

AB = CD = Length  

Let the length be [tex]l[/tex] m.

BC = DA = Width = 4 m

Perimeter of a closed image is equal to the sum of all the sides of the image.

So, perimeter of ABCD:

[tex]p = AB + BC + CD + DA \\\Rightarrow \bold{ p = 2 \times (Length +Width)}[/tex]

[tex]26 = 2 \times (l +4)\\\Rightarrow 2l =26-8\\\Rightarrow \bold{l = 9 m}[/tex]

Residents of four cities are able to vote in an upcoming regional election. A newspaper recently conducted a survey to gauge support for each of the two candidates. The results of the poll are shown in the two-way frequency table below.

Answers

Answer:

3 only

Step-by-step explanation:

Consider the statement, "The two cities with the highest number of respondents, both show more support for candidate A." In the total column, the two highest number of respondents are 471 and 463 which represent Carsonville and Appleton. For Carsonville, the number of respondents who prefer candidate A is 205, which is less than the number of respondents who prefer candidate B, 266. Therefore, this statement is not true.

Consider the statement, "The number of people who support candidate B in Carsonville is twice the number of people who support candidate B in New Thomas." In the table, the number of people who support candidate B in Carsonville is observed to be 266 and the number of people who support candidate B in New Thomas is 138. Since 266 is not equal to twice 138, this statement is not true.

Consider the statement, "More residents of Center City responded to the poll than the number who responded from New Thomas." In the total column, it can be observed that 350 people responded to the poll in Center City and 318 people responded to the poll in New Thomas. Since, 350 is greater than 318, this statement is true.

Consider the statement, "Overall, more residents support candidate A than candidate B." The bottom row of the table represents the total number of responses for each candidate. The number of people supporting candidate A is 797, which is less than the number of people supporting candidate B, 805. So, this statement is not true.

Therefore, the true statement is III only.

More residents of the center city responded to the pole than the number who responded from New Thomas, which is the only correct option. Option B. is correct.


Data given in the table shows the data of elections between two candidates among the different cities.

 

What is Statistic?

Statistics is the study of mathematics that deals with relations between comprehensive data.


I.The two cities with the highest number of respondents both show more support for candidate A. This statement is false because carsonville is the second highest support for A but it does not show more support for candidate A.

II.The number of people who support candidate B in Carsonville is twice the number of people who support candidate B in New Thomas. It is false

III. More residents of Center City responded to the pole than the number who responded from New Thomas. It is true.

IV. Overall, more residents support candidate A than candidate B. it is also false.

Thus, more residents of the center city responded to the pole than the number who responded from New Thomas, which is the only correct option. Option B. is correct.

Learn more about Statistics here:
https://brainly.com/question/23091366

#SPJ5

how many meters are in 250 centimeters

Answers

Answer:

2.5 meters

Step-by-step explanation:

Cases Prudence has a special (cubic) die. The values on its face are the integers from 1 to 6, but they are not arranged ae in a normal die. When Prudence first tosses the die, the sum of the values on the four side faces is 15. In her second toss, the sum of these values is 12. Find what value appears in the face opposite 6 on Prudence’s special die. (Hint: what are possible values for the top and bottom face when the sum of the side faces is 12).

Answers

Answer: 3

Step-by-step explanation:

first, we know that:

1 + 2 + 3 + 4 +5 +6 = 21

Now, which two numbers we should take out in order to have 15?

we can remove the 2 and the 4, or the 1 and the 5.

so here we have two possibilities, 2 and 4 are opposite, or 1 and 5 are opposite (they are located in opposite faces of the die)

in the other arrange, we have that removing two numbers we should get 12.

in order to reach 12, we should remove two numbers that add 9 together.

those can be 4 and 5, or 6 and 3.

Now, notice that in the first restriction we have that:

Or 2 and 4 are opposite,

or 1 and 5 are opposite.

So 4 and 5 can never be opposite, so we should have that 6 and 3 are opposite.

Then we can affirm that the value that appears in the face opposite to the 6, is the 3.

the box plots shows the price for two different brands of shoes​

Answers

Answer:

A. The interquartile range (IQR) for brand A, $10, is less than the IQR for brand B, $25.

Step-by-step explanation:

The most appropriate measure that can be used to compare the SPREAD of the data of the 2 brands plotted on a box plot, is the Interquartile range (IQR).

Interquartile range is the difference between Q3 and Q1.

Q3 is the value which lies at the end of the rectangular box, while the Q1 lies at the beginning of the box.

From the box plot given,

IQR for brand A = 80 - 70 = $10

IQR for brand B = 50 - 25 = $25

Therefore, the correct option is "A. The interquartile range (IQR) for brand A, $10, is less than the IQR for brand B, $25."

log 7 (x^2 + 11) = log 7 15

Answers

Answer:

x = ±2

Step-by-step explanation:

log 7 (x^2 + 11) = log 7 15

We know that log a ( b) = log a(c)   means b =c

x^2 + 11 = 15

Subtract 11 from each side

x^2 = 15-11

x^2 =4

Take the square root of each side

sqrt(x^2) =±sqrt(4)

x = ±2

If f is a function that f(f(x)) = 2x² + 1, which is the value of f(f(f(f(3)))? Please help!

Answers

[tex]f(f(3))=2\cdot3^2+1=19\\f(f(f(f(3))))=2\cdot19^2+1=723[/tex]

Kent Co. manufactures a product that sells for $60.00. Fixed costs are $285,000 and variable costs are $35.00 per unit. Kent can buy a new production machine that will increase fixed costs by $15,900 per year, but will decrease variable costs by $4.50 per unit. What effect would the purchase of the new machine have on Kent's break-even point in units?

Answers

0riginal break even point:

285000/ 60/35 = $166,250

New break even point = new fixed costs / ( selling price - variable cost/ selling price)

New break even point = 285,000 + 15,900. / ( 60-( 35-4.50)/60

300,900 / 60-30.50/60 = $612,000

The new break even point increases.

Calculate, correct to one decimal plice
the acute angle between the lines
3x - 4y + 5 = 0 and 2x + 3y -1 = 0
A. 70.69
B. 50.2
C. 39.8
D. 19.4​

Answers

Answer:

A. 70.69 is the correct answer.

Step-by-step explanation:

Given:

Two lines:

[tex]3x - 4y + 5 = 0 \\2x + 3y -1 = 0[/tex]

To find:

Angle between the two lines = ?

Solution:

Acute Angle between two lines can be found by using the below formula:

[tex]tan \theta = |\dfrac{(m_1 - m_2)}{ (1 + m_1m_2)}|[/tex]

Where [tex]\theta[/tex] is the acute angle between two lines.

[tex]m_1, m_2[/tex] are the slopes of two lines.

Slope of a line represented by [tex]ax+by+c=0[/tex] is given as:

[tex]m = -\dfrac{a}{b }[/tex]

So,

[tex]m_1 = -\dfrac{3}{- 4} = \dfrac{3}{4}[/tex]

[tex]m_2 = -\dfrac{2}{ 3}[/tex]

Putting the values in the formula:

[tex]tan \theta = |\dfrac{(\dfrac{3}{4}- (-\dfrac{2}{3}))}{ (1 + \dfrac{3}{4}\times (-\dfrac{2}{3 }))}|\\\Rightarrow tan \theta = |\dfrac{\dfrac{3}{4}+\dfrac{2}{3}}{ (1 -\dfrac{1}{2})}|\\\Rightarrow tan \theta = |\dfrac{\dfrac{17}{12}}{ \dfrac{1}{2}}|\\\Rightarrow tan \theta = \dfrac{17}{6}\\\Rightarrow \theta = tan^{-1}(\frac{17}{6})\\\Rightarrow \theta = \bold{70.69^\circ}[/tex]

So, correct answer is A. 70.69

Point E lies within rectangle ABCD. If AE = 6, BE = 7, and CE = 8, what is the length of DE?

Answers

Answer:

[tex]\sqrt{51}[/tex] units.

Step-by-step explanation:

Point E is inside a rectangle ABCD.

Please refer to the attached image for the given statement and dimensions.

Given that:

Sides AE = 6 units

BE = 7 units and

CE = 8 units

To find:

DE = ?

Solution:

For a point E inside the rectangle the following property hold true:

[tex]AE^2+CE^2=BE^2+DE^2[/tex]

Putting the given values to find the value of DE:

[tex]6^2+8^2=7^2+DE^2\\\Rightarrow 26+64=49+DE^2\\\Rightarrow DE^2=100-49\\\Rightarrow DE^2=51\\\Rightarrow \bold{DE = \sqrt{51}\ units}[/tex]

if pentagon OPQRS is dilated by a scale factor or ?
from the origin to create O'P'Q'R'S: what is the ordered pair of point S'?

Answers

Answer:

Option (D) : (3.5, 8.75)

If the area of the square is A(s) = s², find the formula for the area as a function of time, and then determine A(s(3)).

Answers

A(t) = 100t^2 + 500t + 625

3,025 square pixels

Answer:

A(t) equals 100t²+ 500t + 625.

The area of the square image after 3 seconds is 3,025 square pixels.

Find the measure of F. A. 44 B. 88 C. 90 D. 46

Answers

Answer:

A. 44º

Step-by-step explanation:

The sum of internal angles in a triangle is equal to 180 degrees, whereas the sum for a square is equal to 360 degrees. Given that three triangles depicted on figure constructs a square, it is to conclude that each is an isosceles triangle. The following relations are presented:

1) [tex]e + 92^{\circ} = 180^{\circ}[/tex] Given

2) [tex]a = b[/tex], [tex]c = d[/tex] Given

3) [tex]a + b + 92^{\circ} = 180^{\circ}[/tex] Given.

4) [tex]c + d + e = 180^{\circ}[/tex] Given.

5) [tex]b + c = 90^{\circ}[/tex] Given.

6) [tex]2\cdot a + 92^{\circ} = 180^{\circ}[/tex] 2) in 3)

7) [tex]a = 44^{\circ}[/tex] Algebra

8) [tex]b = 44^{\circ}[/tex] By 2)

9) [tex]b= f[/tex] Alternate internior angles.

10) [tex]f = 44^{\circ}[/tex] By 8). Result

Hence, the answer is A.

Simplify the following expression. 3x(4x − 3) A. 12x2 + 13x B. 12x2 + 5x C. 12x2 − 5x D. 12x2 − 9x

Answers

Answer:

Multiply using the distributive property.

D is the best answer.

Step-by-step explanation:

The simplified form of expression 3x (4x - 3) is 12x² - 9x.

What is an algebraic expression?

An algebraic expression is consists of variables, numbers with various mathematical operations,

The given expression is,

3x (4x - 3).

Simplify the expression by solving bracket term,

3x × (4x) - 3 x (3x)

12x² - 9x

The given expression can be simplified as 12x² - 9x.

Hence, option (D) is correct.

To know more about Algebraic expression on:

https://brainly.com/question/19245500

#SPJ2

Other Questions
Without the discovery of magnetic reversals recorded on the ocean floor, scientistsA. could not confirm the usefulness of sonarB. could destroy the hypothasis of continental driftC. could not provide a mechanism for moving continentsD. could explain how the seafloor was being destroyed ) A random sample of size 36 is selected from a normally distributed population with a mean of 16 and a standard deviation of 3. What is the probability that the sample mean is somewhere between 15.8 and 16.2 Tariffs Group of answer choices may be imposed either to raise revenue (revenue tariffs) or to shield domestic producers from foreign competition (protective tariffs). are per-unit subsidies designed to promote exports. are excise taxes on goods exported abroad. are also called import quotas. What methods has the state developed to aid them in maintaining power and authority? please help on 3031 Find the missing probability. P(A)=15,P(AB)=1225,P(AB)=7100 ,P(B)=? Blue Nile is an online retailer of diamonds that has used responsive transportation to ship diamonds to customers in the United States, Canada, and several countries in Europe and Asia. Which is the mode of transportation used with this strategy Would you have rathered Clint to die instead of Natasha in Endgame? Why The domain of the following relation has how many elements?[(1/2, 3.14/6), (1/2, 3.14/4), (1/2, 3.14/3), (1/2,3.14/2)]a. 0b. 1c. 4 1/2x-(x-2/3a)+1/4a please help me im so confused! why are goals important for a group discssion Biochemical and genetic experiments have demonstrated that the _________ of tRNA are important for recognition by its cognate aminotransferase-tRNA synthetase. 3. What does the segment suggest to guard against employee misuse of workplace technology? * John painted his most famous work, in his country, in 1930 on composition board with perimeter 101.14 in. If the rectangular painting is 5.43 in. taller than it is wide, find the dimensions of the painting. Haruto Kawa, a Japanese citizen who works for Shin-Ro Corp. in Japan, has been asked to head the company's sales office in the United States. Upon taking the assignment, Haruto will be a(n) _____ manager. Qu dificultades enfrentaron los nacientes estados americanos para organizarse polticamente? The accountant for Mandarin Company is preparing the company's statement of cash flows for the fiscal year just ended. The following information is available:Retained earnings balance at the beginning of the year $949,000Net income for the year 295,000Cash dividends declared for the year 55,000Retained earnings balance at the end of the year 1,397,000Cash dividends payable at the beginning of the year 12,600Cash dividends payable at the end of the year 14,900What is the amount of cash dividends paid that should be reported in the financing section of the statement of cash flows?a. $55,000.b. $57,300.c. $82,500.d. $2,300.e. $52,700. hi , what is third-party companies??? thank HELP!! The aquarium has 6 fewer yellow fish than green fish. 40 percent of the fish are yellow. How many green fish are in the aquarium? Show your work. Examine four factors that promote good child-parent relationship